1
$\begingroup$

Inversion is the transformation defined by $$\hat{I}\hat{x}\hat{I}^{-1}=-\hat{x},$$ $$\hat{I}\hat{k}\hat{I}^{-1}=-\hat{k},$$ $$\hat{I}i\hat{I}^{-1}=i.$$

It is therefore a unitary operator. Let $\hat{H}$ be a Hamiltonian of free fermions on a lattice with sites labeled by a subindex in the fermionic operators. Unitary operators act on fermionic field as linear applications$$\hat{I}\hat\psi_l\hat{I}^{-1}=\sum_l U_{lm}\hat{\psi}_m.$$

Now, from this fact, one has that $$\hat{I}\hat{\psi}_k\hat{I}^{-1}=\frac{1}{\sqrt{L}}\sum_l\hat{I}\hat\psi_l e^{-ikx_l}\hat{I}^{-1}=\frac{1}{\sqrt{L}}\sum_l\hat{I}\hat\psi_l\hat{I}^{-1}e^{-ikx_l}\\=\frac{1}{\sqrt{L}}\sum_{lm}U_{lm}\hat\psi_me^{ikx_l}=\frac{1}{\sqrt L}\sum_{l\Delta}U_\Delta\hat{\psi}_{l+\Delta} e^{ikx_l}=\frac{1}{L}\sum_{l\Delta q}U_\Delta \hat\psi_q e^{i(k-q)x_l}e^{-iq\Delta}\\=\sum_\Delta U_\Delta e^{-ik\Delta}\hat{\psi_k}=U_k\hat{\psi}_k.$$ where I used translational invariance and defined $\Delta=l-m$. However, I understand that parity must act inverting the momentum as $\hat{I}\hat\psi_k\hat{I}^{-1}=U\psi_{-k}$. What is the origin of this difference between the conceptual action of this operator and the result of the above calculation?

——————-

EDIT: I am not asking to solve any exercise, I just have noticed a conceptual contradiction between the result that I obtain and the underlying logic of the transformation.

$\endgroup$
6
  • 1
    $\begingroup$ You are using the symbol $i$ to mean multiple different things. What does it mean in your third equation? $\endgroup$
    – hft
    Commented Nov 22 at 17:20
  • 1
    $\begingroup$ Your $U$ goes from having two indices to having a single index. This notation seems pretty unclear... $\endgroup$
    – hft
    Commented Nov 22 at 17:23
  • $\begingroup$ In the third equation, $i$ is the imaginary unit, meaning that the operator is unitary (i.e. it doesn’t conjugate scalars). When it is used as subscript, it labels a spatial site. I believe this is standard notation and there should be no confusion. $\endgroup$
    – TopoLynch
    Commented Nov 22 at 21:31
  • $\begingroup$ I have made the notation more clear, avoiding the subindex $i$ to denote site labels. $\endgroup$
    – TopoLynch
    Commented Nov 23 at 1:47
  • 1
    $\begingroup$ The matrix $U_{lm}$ is not translationally invariant, i.e. it is not simply a function of $\Delta = l-m$. This is because you pick an origin when you think of inversions. This is also clear from writing down the form of $U_{lm}$. I think that will resolve the contradiction. $\endgroup$ Commented Nov 25 at 21:28

1 Answer 1

1
$\begingroup$

The matrix $𝑈_{𝑙𝑚}$ is not translationally invariant, i.e. it is not simply a function of $Δ=𝑙−𝑚$. This is because you pick an origin when you think of inversions.

This is also clear from writing down the form of $𝑈_{𝑙𝑚}$. If you imagine you have a one-dimensional chain of rotationally symmetric ($s$) orbitals, we can write $U_{lm} = \delta_{l,-m}$. Then your expression becomes \begin{align} \hat{I}\hat{\psi}_k \hat{I}^{-1} &= \frac{1}{\sqrt{L}}\sum_{lm} U_{lm} \hat{\psi}_m e^{i k x_l} \\ &= \frac{1}{\sqrt{L}}\sum_{l} \hat{\psi}_{-l} e^{i k x_l} \\ &= \frac{1}{\sqrt{L}}\sum_{l'} \hat{\psi}_{l'} e^{-i k x_{l'}} \\ &= \hat{\psi}_{-k} \end{align} consistent with the usual expectation.

$\endgroup$

Your Answer

By clicking “Post Your Answer”, you agree to our terms of service and acknowledge you have read our privacy policy.

Not the answer you're looking for? Browse other questions tagged or ask your own question.